Inequality

\(a,b,c\) are positive real numbers and \(abc=1\). Prove that

12+a+12+b+12+c11+a+b+11+b+c+11+c+a\frac{1}{2+a} + \frac{1}{2+b} + \frac{1}{2+c} \geq \frac{1}{1+a+b} + \frac{1}{1+b+c} + \frac{1}{1+c+a}

#Algebra #Inequality

Note by Fahim Shahriar Shakkhor
6 years, 11 months ago

No vote yet
1 vote

  Easy Math Editor

This discussion board is a place to discuss our Daily Challenges and the math and science related to those challenges. Explanations are more than just a solution — they should explain the steps and thinking strategies that you used to obtain the solution. Comments should further the discussion of math and science.

When posting on Brilliant:

  • Use the emojis to react to an explanation, whether you're congratulating a job well done , or just really confused .
  • Ask specific questions about the challenge or the steps in somebody's explanation. Well-posed questions can add a lot to the discussion, but posting "I don't understand!" doesn't help anyone.
  • Try to contribute something new to the discussion, whether it is an extension, generalization or other idea related to the challenge.
  • Stay on topic — we're all here to learn more about math and science, not to hear about your favorite get-rich-quick scheme or current world events.

MarkdownAppears as
*italics* or _italics_ italics
**bold** or __bold__ bold

- bulleted
- list

  • bulleted
  • list

1. numbered
2. list

  1. numbered
  2. list
Note: you must add a full line of space before and after lists for them to show up correctly
paragraph 1

paragraph 2

paragraph 1

paragraph 2

[example link](https://brilliant.org)example link
> This is a quote
This is a quote
    # I indented these lines
    # 4 spaces, and now they show
    # up as a code block.

    print "hello world"
# I indented these lines
# 4 spaces, and now they show
# up as a code block.

print "hello world"
MathAppears as
Remember to wrap math in \( ... \) or \[ ... \] to ensure proper formatting.
2 \times 3 2×3 2 \times 3
2^{34} 234 2^{34}
a_{i-1} ai1 a_{i-1}
\frac{2}{3} 23 \frac{2}{3}
\sqrt{2} 2 \sqrt{2}
\sum_{i=1}^3 i=13 \sum_{i=1}^3
\sin \theta sinθ \sin \theta
\boxed{123} 123 \boxed{123}

Comments

12+a+12+b+12+c11+a+b+11+b+c+11+c+a(2+a)(2+b)+(2+b)(2+c)+(2+a)(2+c)(2+a)(2+b)(2+c)(1+a+b)(1+b+c)+(1+a+b)(1+a+c)+(1+a+c)(1+b+c)(1+a+b)(1+a+c)(1+b+c)(4+2a+2b+ab)+(4+2b+2c+bc)+(4+2a+2c+ac)8+4a+4b+4c+2ab+2ac+2bc+abc(1+a+2b+c+ab+bc+ac+b2)+(1+2a+b+c+ab+bc+ac+a2)+(1+a+b+2c+ab+bc+ac+c2)2(a+b+c)+3(ab+bc+ca)+(a2+b2+c2)+(a+b+c)(ab+bc+ca)12+4(a+b+c)+(ab+bc+ca)9+4(a+b+c)+2(ab+bc+ca)3+4(a+b+c)+3(ab+bc+ca)+(a2+b2+c2)2(a+b+c)+3(ab+bc+ca)+(a2+b2+c2)+(a+b+c)(ab+bc+ca)\dfrac{1}{2+a}+\dfrac{1}{2+b}+\dfrac{1}{2+c} \geq \dfrac{1}{1+a+b}+\dfrac{1}{1+b+c}+\dfrac{1}{1+c+a} \\ \Rightarrow \dfrac{(2+a)(2+b)+(2+b)(2+c)+(2+a)(2+c)}{(2+a)(2+b)(2+c)} \geq \dfrac{(1+a+b)(1+b+c)+(1+a+b)(1+a+c)+(1+a+c)(1+b+c)}{(1+a+b)(1+a+c)(1+b+c)} \\ \Rightarrow \dfrac{ (4+2a+2b+ab)+(4+2b+2c+bc)+(4+2a+2c+ac)}{8+4a+4b+4c+2ab+2ac+2bc+abc} \geq \dfrac{(1+a+2b+c+ab+bc+ac+b^2)+(1+2a+b+c+ab+bc+ac+a^2)+(1+a+b+2c+ab+bc+ac+c^2)}{2(a+b+c)+3(ab+bc+ca)+(a^2+b^2+c^2)+(a+b+c)(ab+bc+ca)} \\ \Rightarrow \dfrac{12+4(a+b+c)+(ab+bc+ca)}{9+4(a+b+c)+2(ab+bc+ca)} \geq \dfrac{3+4(a+b+c)+3(ab+bc+ca)+(a^2+b^2+c^2)}{2(a+b+c)+3(ab+bc+ca)+(a^2+b^2+c^2)+(a+b+c)(ab+bc+ca)}

Let x=a+b+cx=a+b+c and y=ab+bc+caa2+b2+c2=(a+b+c)22(ab+bc+ca)=x22yy=ab+bc+ca \\ a^2+b^2+c^2=(a+b+c)^2-2(ab+bc+ca)=x^2-2y

Putting the values we get,

12+4x+y9+4x+2y3+4x+x2+y2x+y+x2+xy3x2y+xy2+6xy5x2y224x3y270(53x2y5x2)+(43x2y12x)+(xy23y2)+(xy233x)+(xy233y)+(3xy9x)+(3xy27)0\displaystyle \dfrac{12+4x+y}{9+4x+2y} \geq \dfrac{3+4x+x^2+y}{2x+y+x^2+xy} \\\Rightarrow 3x^2y+xy^2+6xy-5x^2-y^2-24x-3y-27 \geq 0 \\\Rightarrow (\dfrac{5}{3}x^2y-5x^2)+(\dfrac{4}{3}x^2y-12x)+(\dfrac{xy^2}{3}-y^2)+(\dfrac{xy^2}{3}-3x) + (\dfrac{xy^2}{3}-3y) + (3xy-9x) + (3xy-27) \geq 0

which is true because x,y3x,y\geq3

Fahim Shahriar Shakkhor - 6 years, 8 months ago
×

Problem Loading...

Note Loading...

Set Loading...